After graduating from Harvard with a degree in biochemistry, I started tutoring and was struck by just how many students insisted that they "didn't get the math gene" or that they "didn't have a natural ear for languages." Coming from a research background, I started going through the journals to see what I could find about the genetics of intelligence. The answer was incredibly surprising. In May 2010, Science published an article about what we know about the genetics of intelligence. So far, we've found three hundred genes for mental retardation and zero genes for above-average intelligence. In other words, we've found no math gene so far. What we have found is that what students believe about their intelligence is an incredibly strong predictor of their behavior and ultimately their academic success. If you believe that you are genetically doomed to fail at math is it any wonder that you don't open your math textbook or ask your teacher for help? The crisis in our educational system is not that students don't have what it takes to succeed. It's that they have everything they need to succeed and they're not using it. Fixing our educational system will not take more books or fixing the teacher's union or even iPads for every kids. It will take destroying a single idea: the idea that people are born smart. Long before there even was an educational system, individuals with few to no resources like Ben Franklin, Abe Lincoln, Frederick Douglass and Booker T. Washington educated themselves by assigning themselves essays, borrowing books from their neighbors and relentlessly self-improving. They understood with blinding clarity what we have forgotten: educations are not given, they are taken.

All too often, we describe today's students as lazy. They are not. They are rational. Based on what they have been lead to believe about intelligence--that it is predetermined by your genes--is it any wonder that they are passive agents in the process? Cultural groups that really believe in the power of practice--many Asian-Americans and many Jews for example--succeed. Mainstream American culture looks at their success and sees it as something we can't copy, because we think their success comes from their genes. It comes from their culture. We are now in a position to carve out an international culture that surpasses anything we have seen before, because it is built on the scientific recognition that the ordinary human brain is flexible enough to become whatever you want it to be. For a long time, we've argued about human potential. Now, we know it...and the news is very, very good.

Along with a friend from college, Katie O'Brien, I'm on a quest to bridge the gap between what people believe about their brains and what science knows about learning. Our first step was to create a fun, cartoon-filled, approachable guide to the latest neuroscience and the exact steps needed to succeed in any subject in school. It's called The Straight-A Conspiracy: A Student's Guide to Ending the Stress of High School and Totally Ruling the World.

Really excited to talk with you all and hear about your own educational experiences, Hunter

Comments: 450 • Responses: 99  • Date: 

someonewrongonthenet205 karma

Indian-American here.

Our success does not come from Indian culture. If you take the average Indian in India and the average American in America, the American would probably be at an advantage.

The Indian kids you see in your country are the descendants of people who were 1) wealthy enough 2) educated enough 3) driven enough to leave their home and start a new life somewhere else. Even if the parents aren't wealthy by American standards, they have an education, usually an MD, PhD, or engineering degree. In fact, if you aren't well educated, you probably won't be granted a visa.

Did you know African immigrants actually do better than Asian immigrants on measures of education? It's probably because they faced more challenges in getting here, creating a very filtered demographic.

On the other hand, take a look at asian groups who were refugees from war (cambodians, laotians, etc). They perform even worse than African Americans.

With the privilege of educated parents, it really shouldn't be surprising that immigrants outperform natives on most measures. Sure, it's "cultural" - but only in the sense that education and wealth give people the cognitive tools to succeed.

My point is, creating that culture that is essentially the product of wealth and education will cost money.

huntermaats23 karma

Is that culture the product of wealth? Or is the wealth the product of that culture? After all, Abraham Lincoln was anything but wealthy? Rockerfeller, Ben Franklin, Andrew Carnegie, Frederick Douglass, Booker T. Washington...this country's history is filled with people who gave themselves world-class educations without coming from or having access to wealth.

I 100% agree that that population--like today's population--was filtered. It was generally made up of people who had enough faith in themselves that if they came half-way around the world, they could build for themselves a better life. (Of course, not all of those people got to make that choice and there are differences in outcome between African-Americans who are the descendants of voluntary immigrants and involuntary immigrants.) Still, the point remains that the culture of empowerment and faith in your own potential does not require money. Why do you need money to look at your mistakes? Why do you need money to go to the library and check out a book? You don't. You need faith in your potential. Faith that regardless of how terrible your circumstances ultimately a human being has the potential to shape their own destiny.

I agree with you. Your success does not come from your Indian culture. It comes from the subculture of educated Indians. Of course, that culture can be gained and it can be lost. Rich parents fail to pass on the values necessary to succeed to their children all the time. Poor parents are able to instill in their children the value and the confidence to thrive.

Why can't make a human culture that internalizes certain universal principles? A faith in the power of practice, the willingness to examine mistakes etc.

someonewrongonthenet85 karma

Is that culture the product of wealth? Or is the wealth the product of that culture?

Both.

In the US, black students consistently report having more positive attitudes towards education, and black people value education more than white people. This is a consistent finding which has baffled sociologists - here is a discussion on the topic, with citations.

Why does a group which values education more underperform? Because it's not about your attitude.

So what are the ingredients to success? Here is a brief list of some of the privileges you need to be born with -

1) feeling safe (physically, emotionally, financially) at home and living a stress free lifestyle. Emotionally troubled kids won't succeed in school.

2) Eating a healthy diet, especially before puberty, and a mild amount of exercise. Getting enough sleep is a big one as well.

3) Pre-natal factors: Unhealthy mothers give birth to low IQ kids. Not much you can do about this once the kid is born, but as a society we can fix this.

4) Growing up in a stimulating environment, especially one which involves you hearing lots of words. If a child doesn't get exposed to a rich vocab early on, verbal IQ takes a heavy hit.

5) General knowledge. Every kid innocently asks "why is the sky blue". Only some kids have parents which can tell them about photons and light scattering. Both my parents are scientists, so for nearly every science related subject I came in with a powerful body of general knowledge that my peers simply lacked. Every kid has that natural curiosity, but my parents could actually satisfy it.

6) One-on-one "tutoring". For example, one summer my grandfather visited from India and taught me algebra, linear equations, etc. As a consequence, I breezed through an entire year of 8th grade math without studying much. Underprivileged kids often don't even have a dad at home, let alone a grandpa who stays at home and teaches them math.

7) Advocacy. Make sure your kid takes the highest level classes. Don't let them slip through the cracks of the school bureaucracy.

All of this is cultural. It technically doesn't cost a dime - in fact, my family probably had less money than yours in the early years...but does your grandfather know how to solve simultaneous linear differential equations? Just growing up in that sort of family causes one to pick up so much knowledge via osmosis. And even though the knowledge was freely given to me, it did cost money to put that knowledge into my parent's and grandparent's heads. All I had to do was express curiosity, and the knowledge would be mine - no trip to the library required!

Poor parents are able to instill in their children the value and the confidence to thrive.

The important thing is not culturally transmitted values and confidence. The important thing is culturally transmitted knowledge and skills.

I don't mean to disregard the importance of values and hard work. My point is that it takes time and money to build a "smart" culture. The first generation must earn enough surplus wealth to send a stable and healthy second generation to college. The second generation must raise the third generation in an enriched environment. Only the third generation will be born with the educational privileges of someone like me (and most likely you as well, if you are reading this comment).

TL:DR: Education itself is a type of wealth, and is handed down from parents to children in much the same way.

Edit: Don't get the wrong idea from what I wrote guys... I think that what Hunter Maats is doing with his book - teaching study skills, reshaping attitudes, etc - is absolutely helpful in the process of acquiring education, and I don't mean to detract from that.

I just disagree with the notion that attitudes and values towards education are the only thing holding our students back. Efforts to create "smart culture" need to be used in addition to public funding, rather than instead of public funding.

huntermaats3 karma

I absolutely agree that "smart culture" needs to be used in addition to public funding. I should clarify what I mean when I say the zero-dollar solution to education. Given the money that we are already spending, what are the things we can be doing to get more value out of that money.

There are very basic behaviors that students don't do that if we were to make them standard parts of our culture would radically transform our educational outcomes. To take an example, look at how different students deal with mistakes. Students who take their mistakes as a sign of their basic intelligence generally avoid them. They shove that bad test in the bottom of their backpack. The result is they repeat those errors. Students who have faith in the flexibility of their intelligence, view those mistakes as fixable. They're much more likely to get that test out, analyze what went wrong and fix it. Standardizing the right behaviors and the most productive assumptions has the opportunity to radically improve student outcomes today without spending another dime. (This does not mean I'm personally opposed to spending more money if it'll make a difference. I just think that if we sit around waiting for money to show up from federal and state governments we may be waiting a very long time. Time that we frankly don't have because the baby boomers are retiring today and my generation and the generation below me need to figure out how to generate a truly staggering amount of wealth to offset the spending on social security and medicare that is coming our way. Again, this is not to blame anyone. It's simply to say this is the state of affairs and say that we need to be looking for every bit of value we can find from our educational dollars. Even if more money was forthcoming from somewhere, shouldn't we be looking to get value of whatever money we end up spending?)

Looking at mistakes and analyzing them to improve is uncontroversial in many areas of life. The FAA isn't perfect but their approach to dealing with accidents demonstrates the power of analyzing mistakes and using those mistakes to create procedures that reduce the likelihood of mistakes. Why wouldn't we standardize that culture in education? Especially, when Anders Ericsson et al have demonstrated that effective practice--deliberate practice--is about focusing on what's not working and using it to improve.

The answer is that until now we've been unwilling to say what a mistake means. Doing poorly on a math test could mean the student didn't use their brain correctly or that the student's brain lacks the capacity to do math. Maybe the student does lack the capacity. Maybe floating out there somewhere in the gene pool, there is a math gene. But we haven't found it yet. And until we find it, we should be looking at what the student is doing and seeing if there are actions that would have prevented those mistakes that can be implemented in the future.

Since we're talking math, let's use math as an example. Math is a set of rules. That's it. We are all capable of following rules, but to able to follow a rule you need to translate it into a recipe for action. You look at three or four parking signs (if you're in LA) and from that you puzzle out can I park here. That puzzling out is all it takes to teach yourself math, but you need the confidence to be willing to sit there and puzzle it. The problem is we've destroyed the confidence in human intelligence, especially when it comes to math. So kids don't do that puzzling out for themselves. They assume they need someone else to do it for them, because math is different from other areas of life and math is hard. And in letting them believe that, we have done all of us a tremendous disservice. Because instead of sitting there with the textbook, looking up the terms they don't know and translating the rules into a clear recipe for action that's as simple as what do I do and when do I do it...they wait for some outside force to solve that problem. If you're lucky enough to be born into a financially or educationally wealthy environment, then that might be a private tutor or grandfather. If you're not lucky enough to be born into some sort of privilege, then you're waiting for the system to change. My point is that there are things you can be doing even if you're not lucky enough to have a grandfather with math knowledge and even if the system doesn't change in the decade or so when you get to go to school.

A paradigm shift has happened in what we know about learning that is on the same magnitude of the paradigm shift that happened when we moved from the "four humors" theory of disease to the germ theory of disease. In disease, that shift lead to a lot of simple behaviors that we all take for granted that save a lot of lives. Sure, there are antibiotics and labs and things that cost money. But, a lot of the benefit, comes from really simple behaviors that we can all do like washing our hands. The amazing thing is that the behaviors that come out of our new knowledge of the brain are much more along the lines of washing our hands in that they rest on the individual having a model for disease that leads them to engage in certain very simple actions.

Standardizing these behaviors need not take money, because innovations like these diffuse culturally. They are peer-to-peer. There are many, many people in the world who I probably won't be able to effectively advocate to. They'll look at my background and say, "What do you know of my experience?" That's why this is something that will require many people to get involved. Whatever subculture you're a part of, you'll be able to advocate to that group and say, "Yes. This is works. I didn't believe it either but it does." The more conversations we have like this, the faster the change will come.

j_one_k37 karma

Why can't make a human culture that internalizes certain universal principles? A faith in the power of practice, the willingness to examine mistakes etc.

Because if you are poor or poorly educated, the power of practice and examining your mistakes is a lot less convincing. If you are poor, you can get screwed out of the rewards of your hard work in a million ways that the rich can buy their way out of. The rich don't flunk tests because they couldn't afford enough food that week. The rich don't live in neighborhoods where the walk to the library runs through a dangerous area. Being poor and having uneducated parents are real disadvantages, even if you can in some cases overcome those disadvantages.

The chance is large that you can tell a kid "practice is powerful" and then have his disadvantage convince him otherwise by depriving him of all the benefits of his hard work. People aren't going to believe your "new culture" if it doesn't reliably work for them, and even a powerful culture isn't going to always overcome the big problems you face if you're poor or don't have educated parents.

Giving famous people as an example is, frankly BS. Ben Franklin was super driven and had a great educational ethic--but he was also super lucky. Telling people "if Ben Franklin can do it, you can do it" may be technically true, but only with a big asterisk: *if you catch the lucky breaks he did.

rmorabia18 karma

Definitely agree with this. OP's system may be a nice fallacy for the rich, but the education system is driven by money. Either you're incredibly unique and driven, or you're rich. The richest kids go to the richest schools, the poorest kids go to the poorest schools, or don't go to school at all. Sure, some break out of the system, but it's rare.

huntermaats2 karma

Isn't that what progress is about? Taking something that is unique and rare and making it widely accessible. Fleming notices that mould kills bacteria. He isolates the cause. Then, we mass produce antibiotics. Well, there are enough of these rare cases that we should be looking at the causes and figure out what we can do to make them happen more often.

Even if you think Ben Franklin's success did derive in large part from luck, wouldn't we all be better off to be super driven and have a great educational ethic? Even if luck is a crucial factor, wouldn't more people capitalize on their luck if they had the right ethic.

My point is given the choice between letting kids make up beliefs about "math genes" and not "getting a natural ear for languages" and actually cluing them into the latest science that demonstrates that practice actually does pay off. Aren't we better off operating on something that is not only more factually accurate but also far more inspiring and motivating?

downneck11 karma

history records only those who were able to rise above, and those people are exceedingly rare.

our country's history may be "filled with people who gave themselves world-class educations without coming from or having access to wealth", but our country isn't.

economic inequality remains a reasonably important factor in our country's educational issues.

notxjack9 karma

this! i can guarantee this guy has never lived around the working poor - some of the luckier students i've worked with in enrichment programs during my college years came from homes where both parents were working over 12 hours per day, and their parents weren't coming home until after 7pm at the earliest. their parents didn't have the skills to help their children with assignments past a 4th or 5th grade level. those less lucky had at most one parent or were moved around to different school systems every few years to live with a new relative because they simply couldn't afford to take care of the child.

tell me again how positive thinking will allow little johnny to bootstrap out of this situation, hunter?

huntermaats3 karma

Positive thinking alone won't. Teaching little johnny how to teach himself might. We usually work with kids who are older than 4th or 5th grade, but send me an e-mail at [email protected]. If you're in or around the LA area, I'd be happy to come meet with the kids and see what I can do to help. If not, maybe there's something else I can do.

ruizscar11 karma

The schooling system is limited by the fact that citizens do not need to be educated any better. They end up sufficient for the jobs that await them.

What does "sufficient" mean in this context? It means you don't need to know much at all beyond basic english and maths. The vast, vast majority of what you learned and promptly forgot -- doesn't matter, really.

The whole point was not to make you remember important things. It was to instill obedience to authority and test your ability to do monotonous, repetitive work in a competitive environment.

famousbirds20 karma

The schooling system is limited by the fact that citizens do not need to be educated any better. They end up sufficient for the jobs that await them.

I'd argue this has never been less true than it is today. The American public education system has never been less equipped to produce capable, employable adults.

However, I'd argue like the OP - that our education system is not a failure because it doesn't teach students enough academic competencies (although that's definitely a factor), but that it fails to teach students critical thinking, self-agency, and basic life skills.

Anecdotal evidence: I volunteered briefly at an alternative high school in Seattle called PSCS. What I saw there was incredible - there was no focus on academic excellence, standardized education/testing, or competition. Instead the focus was on accountability, personal responsibility, community outreach, and following your own passions with support and without criticism.

The results were amazing. Graduating students, for lack of a better term, uniformly "had their shit together". They had practiced setting, following, and realizing their own personal goals, and were amazingly prepared to enter the job market or strike out on their own.

After volunteering, education made sense to me. I had a useless and damaging public high school and college experience, but now work in a field where I've succeeded entirely due to self-education and personal passion. The largest frustration I've experienced is that people think the work I do is complicated, magic, and entirely the province of smart people. The opposite is the case - the work I do is largely dead simple - it's simply a matter of being able to Google and teach yourself the answers to new questions.

huntermaats3 karma

Thanks for mentioning PSCS. I'll check it out.

yourfavnate3 karma

While it that is true, I think you'd have to focus on a broader statistic than naming 6 remarkable individuals. I am with someonewrongonthenet in that those who have "cultures" of trying hard and educating well have often come by that culture either through wealth, or through stringent educational processes (check South Korea for that one) that have both benefits and detracting aspects.

I would certainly like to see the numbers on the average education of wealthy people versus poor people on a Global scale, and I would be willing to put money on the idea that those with wealth are more likely to be higher educated. After that, it would merely be a matter of checking on those numbers for which cultures experience a push for education, and see how the curve changes with them. Do the poor in educationally focused cultures perform to the same level as the rich? What about the middle class? It would make for an interesting sample for sure.

huntermaats3 karma

Although, I've named six remarkably famous individuals, it's important to realize that in their time their choices and their educational outcomes were not so remarkable. In fact, they were remarkably common. See below:

The Americans were a remarkably literate people, and they were even more remarkable in the voracity of their appetites for things to read. Apart from the Bible, of course, which was to be found in nearly every home, the most common reading fare was the newspaper. Cities of consequence were few and far between, but nearly every hamlet of any pretensions had a newspaper. It has been estimated that newspapers went into roughly 40,000 homes on the eve of the Revolution, and possibly twice that number by the end of the century. That figures out at approximately one of every eight or ten families, and when we realize that private copies circulated from neighbor to neighbor and that nearly every coffee house and inn kept files that were open to the public, we may safely estimate that half the adult male population read the newspaper with some frequency. For a colonial population stretched thinly out over a thousand-mile frontier in a raw land, that is an impressively high reading rate; indeed, one might wish that Americans today could and would read so avidly.

Even more impressive is the eighteenth century American's reading of books. As early as 1766 the New York Gazette and Mercury observed that "every lover of his country hath long observed with sacred pleasure, the rapid progress of knowledge in this once howling wilderness, occasioned by the vast importation of books; the many public and private libraries in all parts of the country; the great taste for reading which prevails among people of every rank." The editorialist's enthusiasm was well founded. By the time of the Revolution, nine sizable college libraries existed in British North America, and more than sixty subscription libraries (several of which, like Franklin's Philadelphia Library Company, boasted that tradesmen and mechanics considerably outnumbered "gentlemen" among their users). In addition, a large number of individuals had substantial private libraries that they made available to their neighbors—not merely such well-known bibliophiles as Jefferson, Franklin, and John Adams, but also such others as William Byrd of Westover (who had 3500 volumes) and Robert Carter of Nomini Hall (1500 volumes).

Before we can fight to regain our country's intellectual life, we have to realize that it existed. It's not a pipe dream to think that tradesmen and mechanics should be well read. In fact, if you read "Fur, Fortune and Empire" you'll see that backwoodsmen were often much better read than privileged city folk. That's, in part, because there's nothing else to do out in the middle of nowhere. Of course, that raises another issue. We're all pretty sure we're much busier than our ancestors. For a perspective on that, this is taken from an article in Time Magazine:

"If life seems more rushed than ever, you might be surprised to learn that we Americans don't have less leisure time than we did 40 years ago. We actually have more leisure time, and quite a bit more. What counts as leisure is up for argument, but under every definition the numbers have gone up. We get about 45 minutes a day of extra leisure. Then why does it feel like we have so much less? It might be because we waste half of all our leisure time watching television. The average American adult devotes 2.5 hours a day to this hobby. And for every additional hour we get free, another 30 minutes goes into that boob tube. So if you want more free time, I recommend one thing: turn it off. This is easier said than done, especially during the world series."

As notxjack has said, there are certainly plenty of people who are working every hour of the day to make ends meet. However, there are a lot of individuals for whom this is not true, especially students. The point is not to blame them for how they are using their time, but to look at that time usage and try and understand its causes. A lot of students spend all night doing their homework? Are they using that time effectively? No, because in many cases much of that time "studying" is actually spent worrying about how they're going to do rather than actually learning. For those students who are disengaged from learning, the question is why? Why when we are all hard-wired to love learning--it releases dopamine the same chemical that comes from eating tasty food or having sex--do so many kids despise learning in an academic context? (To be clear, I've never met a kid who outright hates learning. They may like learning sports, dance, video games or languages, but they all like something.) When you dig into the research, you find out that what is poisoning that love of learning is the idea that people are "born smart." It outcompetes a basic, biological drive which if allowed to take its own course would unleash a wave of motivation in our students that would utterly transform our educational system.

The six individuals I've named were part of a much larger intellectual environment that existed after the Enlightenment. That culture believed in the power of human intelligence to be self-improved and believed deeply in the power of human intelligence to solve problems. As David McCullough put it in 1776, "It was a day and age that saw no reason why one could not learn whatever was required—learn virtually anything—by the close study of books…” Was that faith sometimes misplaced or premature? Sure. But, not so misplaced as we might think? The close study of books is all that it takes to teach yourself anything. The Khan Academy and many other websites are now making those resources widely available in a huge variety of formats. (Again, not everybody has internet access, but many, many people do who could be doing much better in school.) They have already provided step one in educational reform. I'm talking about step zero, because before a student...any student rich or poor, young or old, green or yellow...will sit there and study those books closely they need to believe that studying will pay off. In other words, we need to have the right cultural assumptions that practice makes perfect, that analyzing mistakes is essential, that all learning works in the same way.

We can see that culture matters. We can see that cultures can be deliberately changed. (See the diffusion of constructive innovations like germ theory into the public consciousness or the diffusion of destructive innovations like the idea that people are born smart.) Now, we have an opportunity to diffuse three decades worth of research into the public consciousness. As with any diffusion, there will be early adopters and there will be late adopters. Those early adopters may or may not be the sort of low-income families that notxjack is talking about, but I very much hope they will be. Many of America and indeed the world's finest have been poor, smart and hungry. See those 6 remarkable individuals.

karaoke99940 karma

Schools are funded by state and federal dollars. A lot of money rides on students proving that they have learned what they are supposed to learn due to No Child Left Behind. Because of this, state-mandated tests are the end-all, be-all of education.

As an 8th grade math teacher, I have a real problem with this. The pressure is too great at all levels. School districts need the money, principals are under pressure by the district administration to engineer decent scores lest they be demoted, teachers are under pressure by the principal to engineer decent scores lest they be reassigned or fired, and students are under pressure to produce lest they have extra tutoring after school, before school, during lunch, on weekends, and during summer school. A lot of students just say, "Fuck it" and shut down. Why bother trying and failing...why not just choose to fail? At leat then you have some modicum of control over what happens to you. It doesn't sound rational, but that is the mind of a teenager.

There has to be a better way. What do you think needs to be done in this regard?

huntermaats27 karma

I think, ultimately, we're seeing the breakdown of the Industrial-Age model of education. The problem is that one-size fits all doesn't really work in education. It's an inherently individualized process. That's why personal tutoring improves student performance by two standard deviations or, in stats terminology, two sigmas.

The reason why Benjamin Bloom--who discovered this effect--called it the two sigma problem. You can't pay for every kid to have their own personal tutor. The solution is to empower the kid to be their own tutor. The National Academy of Sciences published How People Learn, which--unsurprisingly--summarizes everything we know about how people learn. It all comes down to one word: metacognition. Metacognition means thinking about thinking. Effective learners think about their own thinking. In other words, they tutor themselves. We can give that experience to every kid, but it means a total shift in mindset.

We have to be focused not on force-feeding a narrow curriculum down students throats, but to providing them with the tools and resources to take charge of their own educations. So, here's my plan:

1) Throw out grades and move towards measuring skills. An "A" is totally meaningless. It says you're better than your classmates, but, in an era of grade inflation, it doesn't say you've actually mastered the skill. In a general, grades and degrees mark out people as being special and say nothing about what you can do. That's what actually matters. In an environment that's focused on skills, suddenly competency is no longer scarce. In a grade-based environment, I want you to fail. In a skill-based environment, I may actually want to teach my fellow students because I know that teaching is the best way to learn. I help you and I do better. Some people will feel that way. Some people won't. Either way, it's an improvement.

2) I'd flip the classroom. We're not using the intelligence of our teachers or of our students well. Teachers are delivering the same lectures year in and year out and, frankly, most of this information is in the textbooks or online. The kids can teach the bulk of it to themselves. That frees up the teacher to come in and give individualized feedback where the kid really needs it. Classroom time can be used for customized help and enrichment. Above all, this puts the student at the center of the educational process and gives them a far greater sense of control over what happens to them. That is essential.

karaoke99914 karma

There was talk of flipping the classroom, but it was rejected by the principal. It is a low-SES school, and, when students are irresponsible with textbooks and lose them (we lose about 15 testbooks a year), the school has to eat the cost, and they don't want to do that any more. So, the students don't have textbooks to take home this year. My students can all access their textbooks online, but, being a low-SES school, not all of them are actually able to access it at home, making flipping the classroom unfeasible.

huntermaats10 karma

This is ultimately why I think I bottom-up approach holds more potential than a top-down approach. For these kids, we can't wait for the educational system to change. It will take too long. What we can do today is share with them what we now know about the brain and give them the tools and the confidence to teach themselves.

Urytion6 karma

Have you ever looked into University style schooling? In Australia, we have the Australian Science and Mathematics school. At the ASMS, the school is set up like a University.

Teachers don't follow up your work. It's not in on the due date, you fail. No bells, casual wear, free access to the entire Flinders University campus. Very little to no sheetwork, almost entirely project based. Also, the students are expected to do the vast majority of learning themselves. They're given a list of things to learn, a massive collection of resources, an end goal, and told "do it."

Students there do not take traditional classes (such as Chemistry, English, Legal Studies, etc.).

There are a total of 10 subjects that the 10th and 11th grade students take together. They are: Nanotechnology, Biotechnology, Biodiversity, Earth and Cosmos, Our Technological World, Body in Question, Communication Systems, Sustainable Futures, Order and Chaos, and finally, University Studies.

They do two subjects (on semester rotation), as well as Order and Chaos and University Studies every semester. The two subjects chosen cover the (what we refer to as) the Big Three (Biology, Chemistry, and Physics), Maths, and English, giving a full scientific education on 3 subjects. The reasoning is that (for example) Biology and Chemistry don't stand wholly apart, despite what traditional schooling teaches. Order and Chaos focuses primarily on Mathematics, but also contains scientific ethics, and research procedures.

University studies is a program where any teacher, or volunteer from the community can teach a class of their choosing. Some of these include Music, Photography, Film Making, Paramedic Pathways, Game Design, and Cryptography.

Students do not necessarily have to be high achievers. They just need to be passionate learners, however, it does tend to attract high achievers anyway.

huntermaats2 karma

Very interesting. Did you attend ASMS? Do you teach there?

answeReddit34 karma

It seems 'politically incorrect' today to suggest that some cultures are superior to others, but cultures that emphasize practice empirically seem to serve their members better.

Have you gotten any accusations of racism for drawing a cause and effect between culture and academic success and if so how do you usually respond?

huntermaats42 karma

Not really, but if I did here's what I'd say.

We have come to confuse culture and race. They are two entirely different things. For example, although most Asian-American groups do well in school, not all of them do. Notably, the Hmong people don't. That's because their culture is different. The whole idea of race is in general pretty ridiculous from a biological standpoint. The genetic differences between human beings are so vanishingly small, that if we want to understand the differences between groups we have to look somewhere else.

There's now a very solid body of research on the difference that culture makes in educational outcomes. There was a great review recently of some of this research on NPR entitled "Struggle For Smarts? How Eastern And Western Cultures Tackle Learning."

So, that culture makes a difference is really not debatable anymore. The important thing to realize is that we should break cultures down into their component parts and steal for ourselves the parts that are good. It is unquestionable that individuals who are willing to analyze their mistakes to improve their process will do better. Is that approach entirely present or entirely absent in any culture? No. But it does predominate in certain cultures. The more any culture can internalize that attitude the better they will do.

The whole issue is that confusing of "race" and culture. You don't choose your skin color. You do choose what you believe about intelligence. And, as an individual, you shouldn't buy wholesale into what any group believes about anything. You should make your own determinations about what your values are. In the process, you will help shape the values of those around you and we can ultimately--by individuals improving themselves--begin to shape a human culture that values certain core principles:

1) The willingness to examine your mistakes. 2) Deep-seated confidence in the power of practice. 3) A voracious desire to learn as much as possible about everything and anything.

This culture is empirically and objectively better for mental health, educational and economic outcomes. The language(s) you choose to speak, your preferred forms of art and to a certain extent your cuisine...that's your choice and the appropriate place for cultural relativism. Assuming that your brain is flexible enough to learn anything, that's not relative. It describes reality.

The key is reiterating that culture is not race and to be clear about which aspects of culture are objectively better and which aspects of culture are subjective preferences.

Gddf5 karma

No cultural group is superior in general, but some are better at certain things

huntermaats11 karma

Exactly. Why wouldn't we deliberately create an intellectual culture that draws on the best of each?

trannybonaduce33 karma

As a student in public school, about all that got me through the day and engage were the arts and music programs. People need to be able to express themselves. My take on academic apathy is that you can't cure it without giving someone a reason to care. What impact do you think the current trend of gutting arts education and music education will have on the next generation of students?

trashed1314 karma

I second this Hunter. Could you address this? My mother works her ass off at three part time art jobs, and then has to essentially babysit the kids. The schools are cutting art. The NCLB and Race to the Top aren't helping.

buppers8 karma

maybe the apathy is part of the problem of nobody caring because they think it's "too hard". Also when kids decide they don't have the internal ability to do something, they belittle that thing and everyone decides it's 'useless' knowledge anyway. Maybe we would have better luck if we could frame educational topics in a way that's relevant to a student's life.

huntermaats20 karma

The real issue is that we've compartmentalized intelligence. We think that there are "math people" and "English people" and "arts people" and that if you are good in one area, then you must be bad at another. The artists of the Renaissance were also inventors. We should be encouraging our kids to become good at as many things as possible, because you'll need it all.

Personally, when I was in high school, I thought of myself as a science person. As buppers says, "I thought I was "bad" at writing and art and in reaction to that I belittled them." Well, that was a really self-defeating move on my part. Because, it turns out that what I wanted to do was get the latest science into the hands of people everywhere. That meant I need to know how to write and I needed to know how to do graphic design. So many things today require it all: science, language and arts. So, if you gut arts and music, you'll end up with more of what we already have scientists who are terrible at communicating their ideas and communicators who are intimidated by science. We should be aiming to create Renaissance men and women. Apple is as successful as it is as much for its design as its technology.

karaoke99917 karma

I have a theory that instant gratification is a major component to student apathy.

As a child (I am 36 now), when I wanted information, I had to go to the library, look up the topic in the card catalogue, find the correct book, look in the table of contents or the index to figure out where in the book to look, and THEN read it to get the information. Today, kids can wikipedia and quick text search to get just about whatever information they need.

When I wanted to watch a TV show, I'd have to be in front of the television the moment it was airing, sit through the commercials, all the way through until the end of the half-hour (or hour), and do that week after week just to find out how it all ends. Today, kids still have that option, but they can watch the shows online now as well, the next day or the next week or three weeks later, or they can wait until the entire season is on Netflix or Hulu and binge-watch them commercial-free, and YouTube search their favorite moments.

In my opinion, there is an expectation among today's youth that they can have whatever they want, whenever they want it, and, if they can't get it right away, it isn't worth it. Thus, when they don't understand a topic in math (for example, since I teach 8th grade math and Algebra I), they get frustrated and shut down.

Your thoughts?

huntermaats17 karma

Expectations play a huge part in our persistence, so, in that sense, I think you're absolutely right. The thing is that expectations are very area specific. We've met plenty of kids who have about a half-second tolerance for any difficulty in solving a math problem, but who will chip away for hours at a drawing or a video game. The problem is that the myths we hear about geniuses--that Mozart was born composing masterpieces and that Newton discovered gravity--and the myths we create about our own classmates--that the kid in the first row gets straight-A's without studying--that destroy our persistence in school. So, I think you're absolutely right that many students expect instant-gratification in math...and that's a huge problem. Not sure to what degree there's cross over in our expectations from other areas of life.

GnothiSeauton_Fool17 karma

It seems misleading to claim that intelligence is 0% heritable. Pardon for the Wikipedia citation (it's a good overview and has peer-reviewed citations), but the evidence I've seen concludes that intelligence is probably around 50% heritable, if not more. Not to say that we should teach students to focus on heritability of intelligence, but it doesn't seem ethical to spread misinformation either.

What you're saying clearly has merit though. If I recall correctly, one of the greatest predictors of success in the Israeli Defense Forces was if the teachers/leaders believed that the recruit had high potential. The Pygmalion effect definitely exists, and is significant. However, in the original Pygmalion experiment, they were only able to get at best .3 standard deviations above the mean, with mean effect of .16 standard deviations. While that's incredibly significant, it's by no means a silver bullet.

Does a neuroscience approach give a strong enough causal link to empirical results to draw reliable and actionable conclusions? In practice, is it really a zero-dollar solution? And is the effect strong enough to "fix our educational system" as you claim?

EDIT/PS: I'm just exceptionally wary of anything that talks about a conspiracy where "the truth has been kept from you" and implies a catch-all solution to your problems.

huntermaats6 karma

In testing out titles on our students, we found the conspiracy title worked best. Like everything else we've done, it all comes down to what works best for students.

We're not claiming that intelligence is 0% heritable. We are asserting that no specific gene linked to above average intelligence has ever been found. The thing that we do know from extensive research see Dweck et al. is that what you believe about your intelligence has a huge effect on your persistence and ultimately on your outcomes.

The Pygmalion effect is definitely part of it, but it's not the only effect. It's about what the teacher believes, but also about what the student believes. On top of that, the second half of the book is about teaching students how to think about their own thinking, metacognition. That's the thing that National Academy of Sciences called the most distinguishing characteristic of effective learners. Essentially, what you're doing there is tapping into the two sigma effect of Benjamin Bloom. (Bloom found that private tutors improve students performance by two-standard deviations. Really, that effect is from checking in to see whether the student understands at each stage. Effective students already do this for themselves. Why not give that experience to every kid?)

In our experience, we've been able to combine all of these many effects together in order to create a process that consistently shifts kids to getting straight-A's. We've distilled that into a book and are now hearing from people all over the country who have been able to implement that and achieve the same effect. Now, did they get it for zero dollars? No. They got it for $14.49. That said, it's at zero dollars to the tax payer. And, frankly, if no-one ever bought another copy of the book and what we're saying spread purely by word of mouth...I'd be a very, very happy camper.

It can cost zero dollars. And I think it can fix our educational system. If I'm wrong, I'll buy a hat and eat it.

hostergaard3 karma

We are asserting that no specific gene linked to above average intelligence has ever been found.

Which says nothing at all. Absence of evidence is not the same as evidence of absence. By asserting that no genes have been found that is linked to above average intelligence you are only insinuating that intelligence is not heritable.

Have you considered that the difference you experience in a students success depending on their persistence is not due to intelligence non-heritable but rather that education in school is made to be so simple that even the dumbest person can grasp it and thus persistence rather than intelligence makes the difference?

huntermaats3 karma

Absence of evidence is not evidence of absence. But in an absence of evidence, the certainty with which many students believe that they "didn't get the math gene" or that they "don't have a natural ear for languages" and therefore don't persist is unjustified. That's the purpose of referencing the conclusions cited in my original post. To say that the certainty of students is misplaced and ultimately corrupts every aspect of the educational process.

Students look at their test grades and infer genetic causes. That's ridiculous. We're currently allowing our students to form outlandish conclusions about their intelligence and we don't have to because there are certain things we know that are empirically demonstrated to produce positive outcomes. Regardless of as-yet-undiscovered genes, there is plenty of evidence on neuroplasticity, heuristics, automaticity, attention, emotions and the acquisition of expertise that we can and should be equipping our students with. We don't know if a math gene exists. We do know that telling students they don't have ability is damaging to performance. We also know that telling students they do have ability can be damaging to performance. See below:

http://www.mendeley.com/catalog/praise-intelligence-undermine-childrens-motivation-performance/

So the question is where do we go from here? What would you advocate?

As for your second question, the real question is what does simple mean? We now know that learning is about the interplay between automaticity and attention. If you do something often enough, it becomes automatic. Putting your attention on things is how you make them automatic. This is how people learn to read or master any other skill. You put your attention on your A,B,C's until they're so automatic you don't have to think about them. That frees up your attention to focus on whole words. You automate those which frees up your attention to focus on clusters of words. Eventually, you can so thoroughly automate the mechanics of reading that your attention is freed up to think about meaning and imagery. Automaticity is how you free up your attention to handle complex tasks. If you do those often enough, they become simple and then you can do things that are more complicated.

A big part of the issue is that students are using their attention very, very differently. Some of those approaches are more effective than others. For example, much of mental math is about using the commutative property. Students don't see that approach being used. They just see that the other kid knows the answer right away. Getting students and teachers to talk out loud as they work through problems is very helpful, because it allows everybody to see how the person is breaking down the problem in their head. What heuristics are they using? Are there better heuristics etc?

Imwe14 karma

In my education I noticed a big difference between people who liked to learn for the sake of learning and others who were only interested in learning enough to pass tests. The former still read books for fun while the latter don't. What is something that will help schools produce curious students instead of test passers?

huntermaats6 karma

It's the difference between external and internal motivation. Test passers are only interested in the grade. People who learn for the sake of learning aren't looking for a reward other than learning itself. If you look at the work of Dweck and others, it's really about that growth mindset rather than the fixed mindset. People who believe that their brain can always change become curious students. People who believe that their intelligence is fixed tend to become test passers who are only interested in doing the minimum amount. So, yeah, it's about giving kids the perspective that this is the brain you will have for life and the more amazing things it will be capable of doing. Because, in the real world, the big rewards are not just for clearing hurdles, but from always trying to figure out how you can take what you're doing further.

nsgafc14 karma

I really, really, truly, fucking hate Algebra. Unfortunately, I need to get past it to attempt to earn my degree. Do you have any specific ways to help neuter any prejudice against it?

huntermaats29 karma

Fortunately, you've correctly identified half the problem: you "hate" math. I'm sure you do hate math, but the issue there is emotional. That's half of how you can make your experience a) less painful and b) more successful. You don't have to love math, but if you can at least move from really, really, truly, fucking hating Algebra to being neutral on math that'll free up your attention to engage with the math. That's the key. Your emotions drive your attention. When you love something, that emotion drives your attention towards the subject. When you hate it, you avoid it by the plague. That's not news to you. The point is that the problem is not math. The problem is your emotional response to math. Anything you can do to move that--will allow you to beat this class. Even saying, "I hate you math and that's why I'm going to beat you into the ground." That'll get you engaged. The second part is understanding the issue with algebra. Algebra is only rules. Rules can be clear enough to act on and they can be fuzzy. You look at a parking sign and for the first few seconds you don't know what to do. Can I park here or not? You then sit there until you have a "recipe for action." At this time on a Tuesday, I can park here. That's all algebra is: a bunch of parking signs. For every piece of algebra, you need to clarify each rule until you've answered two things: 1) what do I do? 2) when do I do it? In the book, we use the analogy of Betty Crocker. She takes a recipe and distills it down into three easy steps. That's possible for anything in algebra. You can feel however you want about parking and baking a Betty Crocker cake, but there's no doubt that they're doable. You can do this. Beat algebra into the ground one rule at a time.

nsgafc13 karma

Yeah, I'm buying the book now.

huntermaats6 karma

Let me know what you think and if you have any other questions.

dmetvt11 karma

I love the idea, but I'm curious what you would suggest about how to implement it. Are there policy changes or other top-down approaches that you could see helping encourage a culture change or does this require a huge amount of bottom-up coordination? Your book is a cool idea, but I'm afraid that the only people who would be interested in reading it are already motivated enough that they would find another way to succeed. How can you get the message across to kids who have already fallen behind?

tejon11 karma

On a related note, applying the term "zero-dollar" to this topic seems either painfully naive or deliberately disingenuous -- neither of which encourages faith in the message.

huntermaats4 karma

It may be painfully naive. Time will tell. That said, I actually think it may be the zero-dollar solution. Zero-dollar solution does not mean it will come with no cost. It will require each of us to think, reflect on and change what we believe. We've gotten so used to the idea that effecting change requires money that we have lost sight of the fact that more often than not it requires a change in attitude. Of course, I'm sure there are great interventions you could design that would cost money, but part of the idea we have to break is the idea that more spending leads to better results. It doesn't necessarily. We've spent money on textbooks and teachers already. Do we have to spend money to get them to use them too? Why do some students use them and some students don't? Isn't that just an attitude?

CommentsRarely6 karma

Can you name any practical steps we can take towards beginning to realize your dream for this cultural approach to education? As someone who has worked around educational policy, I have to tell you that this sounds equivalent to saying that all we must do to eliminate problems that stem from socioeconomic inequality is to convince everyone to not be greedy anymore.

huntermaats2 karma

1) Show students that the brain can change. 2) Encourage students to be specific about their mistakes. Saying "I'm bad at math" is meaningless. Saying "I do not know what an exponent is" is specific and fixable. Being specific about what you do know, what you don't know and not moving forward until you have fixed it is a really simple behavior that is easy to institutionalize in a culture. 3) Students need access to their tests after they take them. It's unacceptable to not give students the opportunity to look at their mistakes and have time to analyze them until they have developed a specific course of action for preventing that mistake from happening again. This is what the FAA does with accident investigation and the FAA isn't perfect but their response to crashes would go a long way on its own to fixing our educational system if we institutionalized that culture.

This is all really about specifically identifying what's not working and stopping until you have a recipe to fix it.

Specifically, getting something wrong on a test is a very specific piece of feedback: it means you either broke a rule or a fact. If it's a fact, you learn it...see below. If it's a rule, the goal is not to memorize it but to turn it into something actionable. Every rule can be transformed into something as simple as "what do I do" and "when do I do it." If our culture assumes this logic--essentially uses these heuristics or mental shortcuts--then students will save a huge amount of time and get far, far better results.

Another big issue is the approach to memorization. The general assumption and this is fairly international is that if you need to memorize something it's time to get out the flashcards. Well, it turns out that repetition--while it does work--is the least effective and the most boring method of repetition. Instead, students should learn to look for logic. Why does this word make sense? It might be by looking at the etymology. It might be by finding a made up logic. Finding "memory hooks" is not news. Spelling bee champs tend to approach their task that way. That's why they're champs. Furthermore, the role of emotion in memory is huge. Things that are funny, emotionally-relevant are disgusting are instantly memorable. Internalize that understanding in our culture--instead of the understanding that some people have naturally good memories--and you'll radically improve outcomes. The final part of the memory picture is prior knowledge. The more you know, the easier it is to memorize knew things. This runs counter to what most people assume. They operate on the idea that human memory works like computer memory and that the goal should be to limit the amount we learn. That's part of why students cram for tests. The problem is that the flashcard approach to learning turns learning into an uphill battle. Students are retaining very little information and so they end up having nothing to connect the next set of test material to. So, yes, if, for example, you change our cultural assumptions about how human memory works, people will act differently they'll have more rewarding experience emotionally and they'll do better.

The point is that these are choices that individuals can choose to implement for themselves. It would be great to see programs that support these, but we don't have to wait until they show up.

Essentially, students are diagnosticians. They look at their performance all the time. What did I know? How fast did I know it? How did I do on the test? How did everyone else do? Based on these "symptoms", they reach "diagnoses." These diagnoses are picked up by means of culture. We want to create an intellectual environment in which the default cultural assumptions motivate students to action rather than throwing their arms up in despair.

What do you think?

buppers3 karma

This is a good question - Are there plans to do any in-school presentations? I know that similar interventions have been done with positive effects (don't remember the authors, but they explained how learning works in the brain to the students, etc)

huntermaats7 karma

We'd love to see top-down action, but I think part of the issue is that we've been waiting for that top-down action for a long time. There's an inherent problem in this in a democracy, because ultimately change from the top has to fit with what the people understand. So, either way, I think it ends up being bottom-up. Carol Dweck has already been doing work to put neuroscience in the hands of kids (probably who buppers is referencing). Check out her site to that effect:

http://www.mindsetworks.com/default.aspx

We've also been doing in-school presentations and talks to adults. We plan to do many more. The goal is simply to move the mindset wherever we can. There are pockets of people all over the world who know about this stuff from Dweck, Gladwell, us and others. Ultimately, all of these ideas will coalesce into a common understanding from which we can operate.

Those early adopters probably aren't going to be the kids who are turned off to learning. The issue is cultural and the change will be cultural. If I'm a kid who is turned off to learning, I probably wouldn't listen to me. But, for example, the rapper Macklemore wrote a song that featured the 10,000 hour rule. That idea has now reached a whole bunch of kids who probably wouldn't be our early adopters. One of the things I want to figure out is how to get the UFC fighters to talk about how the practice they do to win fights is the same that is required to master any skill. That's another cultural group. The book is just one thing. Other people will take these ideas and repackage them for their own subculture.

ThePaisleyChair10 karma

I'm curious. I'm sold on the idea that we often see a defeatist "I can't" rather than "I don't know how" attitude. The more a student thinks that s/he cannot do something, the harder it will be for him/her to do it. That I get.

But I think you're missing some larger issues that make the education system work for some and not for others. Poverty rates among Asian children is much lower than the national average, for example. Students living in the zone for a failing school don't always have the opportunity to get into a better school. The two "model minorities" you've cherry picked here tend to have better access to resources like educated community members, wealth, and political power, as well. How do you account for issues other than "believing they can" in your "it comes from their culture" argument? Things like poverty, location, incarceration rates, job opportunities?

Have you talked with people trained in education research, pedagogy, or child psychology? Do you work with teachers?

Don't get me wrong, please. I'm not attacking what you're doing. I think it's great to bring an awareness that all students can. But I'm a little concerned that your selling tactics present this as a panacea for an enormous problem and that by doing so, it A) discounts enormous obstacles and says "Oh you can if you just try!," B) sets your system up for not doing as well as it could by ignoring outside factors, and C) comes across as uninformed and naive.

huntermaats2 karma

Have you talked with people trained in education research, pedagogy, or child psychology? Do you work with teachers?

Yes, to all of the above.

We're not discounting those obstacles. Many of them are huge and I would love it if every kid could go to big, beautiful, clean, well-resourced school. The problem is what do you tell kids in school today. By the time the system changes itself, these kids will likely be in their 60s. Whoops. The point is that it is possible to succeed in an imperfect environment. That after all is what makes humanity remarkable. Many people around the world have given themselves world-class educations with less-than-world class resources. That's because the most important thing in an education isn't an iPad. It's a human brain. It's up to you to apply it, but before you feel motivated to try you have to get woken up to how remarkable it is. We're not ignoring those outside factors. We'd love for them to change. It's just that they don't seem to be. So, assuming those outside factors remain the same for the forseeable future, what can we do to make the most of everybody's intelligence. What is the cheapest, fastest thing we can do?

I'm not sure if you've heard of William Kamkwamba, but he was a kid in rural Africa who while living through a famine taught himself how to build an electricity-generating, wind-turbine from junk he found in a junkyard and a book. Outside factors don't get much more daunting than that, but he found a way to take an education no matter what. That's because William had faith in his intelligence. He kept at it. His success didn't come from resources, it came from resourcefulness. Let's take that attitude and share it with the world.

arian48710 karma

I'm a Math major and have dabbled in tutoring and the comment I get very often is: "I'm terrible at Math! I just can't do it!" They say this enthusiastically as if it isn't a problem and this to me is the biggest problem in our society. It's 'okay' to be bad at math, and it isn't instilled from the kids at a young age that it ISN'T okay. If someone said something similar about reading they would be ridiculed and it becomes a huge problem. "I'm terrible at reading! I don't get it!" would be met with a different tone than the math equivalent.

My question is, what needs to happen for this societal shift to take place? It's about fucking time we started seeing Math as just another language and necessary skill, as opposed to an optional thing.

huntermaats2 karma

Couldn't agree with you more. It's a huge problem and one of which we're already facing the consequences. It's not just the lack of engineers, mathematicians and scientists limiting our economy. It's a large section of the voting public that is deeply uncomfortable with numbers and science. That's a real problem.

Cultures are made up of individuals. Change individuals and at a certain point the culture will shift which will change the rest of the individuals.

Fortunately, the science is on your side. We now know that learning one thing is the same as learning anything. Math is just another language and a necessary skill. So, go for it. Spread the word. Write op-eds. Make a YouTube video. Write a book. Do a Reddit AMA yourself. Have you seen Danica McKellar's books? They're one attempt to get people past their fear of math. You might want a more tough-love approach. The point is we need as many different voices out there as possible.

What are you planning to do after graduation?

gigaflar37 karma

My seven year to old daughter was kicking ass at math all through first grade. Suddenly in 2nd she has picked up the idea that math is hard. I dunno if it was from friends, teachers, or other adults but I've been have a hell of a time countering this with something useful. I can't believe it has started so early. Any suggestions? I remind her all the time just how capable and smart she is, but it isn't always enough.

hclchicken9 karma

Don't praise the person, praise the performance. This is how all management and parenting experts state should help your kids.

An example, telling your kid that they are good or bad is like telling them that they can't change how they combat a problem. If you praise how they are trying then they will do their best.

There was an NPR segment on this and in it the researchers gave different societies the same impossible math problem. When given to american kids they tried for <1 minute before giving up. When given to Japanese kids they had to stop them at the one our max because they hadn't given up. These kids who think they are smart view it as a shortcut.

huntermaats9 karma

The NPR piece hcichicken is referring to is "Struggle For Smarts? How Eastern And Western Cultures Tackle Learning." It's really excellent. You can find it here:

http://www.npr.org/blogs/health/2012/11/12/164793058/struggle-for-smarts-how-eastern-and-western-cultures-tackle-learning

It's excellent. Thanks for mentioning it.

huntermaats7 karma

Unfortunately, the idea that girls are "bad at math" is pretty prevalent in our culture. It's totally possible, she picked it up from friends, TV shows AND, sadly, teachers. We even encountered one teacher who divided up her class into the "hate math group" and the "love math group." Ugh!

Katie, my co-author, and I have found that debating the value or doability of a particular subject is a quagmire that is very, very hard to get out of. The key is circumventing the idea of subjects being different at all. The point is all learning works in the same way. This means that instead of focusing on a problem, you can figure out how to draw from strength. What does your daughter do well in? What does she love? How can she use what she does there to improve her experience of math? Every kid wants to do well in every subject. Your daughter, in spite of what she might say, really wants to be a rockstar. You might find it easier if you approach it from the perspective of "since you already know how to learn one thing...you know how to learn everything. You just need to do in math what you do in everything else. Break it down and clear up the pieces."

Another big thing that we know understand is that it's always better to frame success in terms of actions than ability. Part of the issue may be that you're telling her how smart she is. She may be looking at her own in-born intelligence to understand her success and failure. The problem is that when she then doesn't succeed, she's stuck. You might enjoy the book "Mindset" by Carol Dweck. It's pretty awesome. That's really what it's all about. Moving her thinking away from the idea that people are smart to the idea that anyone can become smart if they do the right things.

I'm not sure if she's in a place for this. But the most helpful thing is sitting down and just analyzing her results. Why did the mistakes happen? What can be done to prevent them? That's really what's going to give your daughter the confidence to take charge and succeed in any subject. Seeing that success comes not from her, but from the choices she makes.

Does that make sense? Let me know if you have any other questions, Hunter

the_ouskull5 karma

As a teacher, I'm here to tell you I agree with you. But, if you think that existing knowledge is going to change the culture of hopelessness for the majority of struggling students, it's because you're thinking more like an educator than a sociologist.

Example... next year, states are switching from the abject failure of NCLB to Common Core. How do you feel this switch is going go work? What can/may/will result from it?

huntermaats4 karma

It's pretty hard to get worse than NCLB. I think the problem is we need to make that culture shift. Kids are so turned off to learning that we're trying to force-feed them some small, sub-set of information. We tell ourselves, "If only, we could get them to learn the three R's" etc.

There's a great story that MLK was at a conference and a speaker was talking about how if only you could teach kids to read, then everything else would be possible. MLK leaned over to the guy next to him and said that first you have to teach them to believe in themselves. Debating curriculum is worthwhile. But, I think it's step one and unfortunately we're still stuck on step zero: giving kids the confidence that they can learn.

Sharing the existing knowledge is going to be the first step. Other changes will follow. People's value system will change. Imagine if learning became outright cool. If the people on TV were showing off their brains instead of their booties. It's not just students who have lost faith that that is possible, it's a lot of adults. We've gotten so used to the current depressing state of affairs that we've lost sight of the culture that much of the United States had for the first hundred years of its life. Why can't we return to and surpass the beliefs that gave us the Ben Franklins, Booker T. Washingtons, Abe Lincolns, Henry Knox's and Frederick Douglass's? In David McCullough's 1776, there's a great line that sums up the intellectual beliefs of the time, "It was a day and age that saw no reason why one could not learn whatever was required—learn virtually anything—by the close study of books…” If we get the culture right, then we won't have to debate what small sub-set of information students must learn, because these kids will be voraciously teaching themselves...the way most Americans used to and some Americans still do today.

In other words, I think NCLB and Common Core are variations on a theme. The content may have changed. The testing may be different. The premise, though, is the same.

karaoke9995 karma

In other words, I think NCLB and Common Core are variations on a theme. The content may have changed. The testing may be different. The premise, though, is the same.

Exactly. The mentality of bureaucrats is that students just need to be challenged, given more real-world applications of math (for example, since you used it before), and held to higher standards. That is what, in their minds, will improve learning. And you and I understand that "you can lead a horse to water, but you man't make it drink" is as true with children as it is with horses.

huntermaats7 karma

Or as Dorothy Parker said when asked to use horticulture correctly in a sentence. "You can lead a whore to culture, but you can't make it think." What is true of horses is true of whores is true of children.

huntermaats4 karma

Until you kids have confidence in their own intelligence, they won't think. If they're not thinking, then that's a problem.

the_ouskull3 karma

Until kids have the motivation to think, they won't think. But a third-generation welfare child with no father and not much of a mother (poor, uneducated - generally, WAY the fuck over her head) has NO motivation to better themselves.

Mother is the name of God on the lips of every child. When your God is a model for academic indifference, you're not too into church. THAT culture can't be fixed. I truly feel that dysgenics has already pushed our low SES communities past Gladwell's proverbial tipping point and into a permanent cycle of hopelessness.

huntermaats3 karma

And they won't have a motivation to think until they believe that thinking pays off.

derpinita5 karma

What are your thoughts about ADHD?

huntermaats8 karma

I've known students and parents who say that an ADHD diagnosis saved their lives and I've known students and parents who say that an ADHD diagnosis was the worst thing that ever happened to them.

I think the really interesting question is, "Why is there a debate about ADHD?" Nobody is debating the existence of cystic fibrosis. (I mean...we live in the age of the internet, so I'm sure someone is.) The problem with ADHD and IQ and any of these assessments is that the objective data does not necessarily lead to the cause.

Nobody doubts that kids are inattentive, but the question is what is the cause of that inattention is it psychological (boredom, self-concept, information is not connected to personal goals) or is it chemical (ADHD-induced by diet and exercise) or is it genetic (ADHD-induced by genes)?

My point on ADHD is we should embrace that doubt and consider all those causes. Exercising and eating better isn't going to hurt. Personally, from the practical standpoint of being an educator, I'm not going to allow ADHD to let me off the hook as a teacher.

I have worked with lots and lots of kids who have been diagnosed with ADHD. I have never met one who is not capable of intense focus when they're doing something that interests them.

Rather than say, "This kid has ADHD." I say, "Woah! I'm competing for this kid's attention and being outcompeted by a squirrel. That is sad, sad news. Better figure out a way to engage their attention." Partly, that's about holding them accountable for paying attention. Everybody's mind wanders, but everyone can bring their attention back. And every teacher is sometimes boring, but what we're learning is pretty amazing...when presented the right way.

ADHD. Is it overdiagnosed? Is it underdiagnosed? Does it exist? Do we all have it? I don't know. I do know it's my responsibility to engage the student no matter what.

ministryofsound5 karma

"All too often, we describe today's students as lazy. They are not."
when is the last time you've been in a highschool dude..

Untjosh12 karma

As a high school math teacher I can confirm this. There are a lot of lazy students, and from my experience half of the time they are lazy because they aren't held accountable at home to do their best in school. Not sure how this system is going to address that reality.

huntermaats2 karma

The research shows that those ideas that people are born smart are what kill motivation. If you think you'll never be good at math and you think that you're being pointlessly forced to do it for other people, is it any wonder that those students aren't motivated? In our experience, you get the idea of being born smart out of the equation and put the student in charge of their own learning and you find that the motivation recovers.

corey15055 karma

Are you familiar with Carol Dweck's work on mindset? How does your approach differ?

huntermaats1 karma

Our work is heavily grounded in Carol Dweck's work. We're huge, huge fans of hers. All we've done is draw together the work of Dweck, Ericsson, Kahneman and a whole bunch of others and synthesize it into a form that even the most disgruntled teenager can use.

WitOfTheIrish3 karma

  1. Based on a lot of your responses, I would love to know what you think of the Montessori method of running schools.

  2. Does your book, or your research, do much to tackle the issue of overcoming generational learning apathy/deficiencies (i.e. parents not just not being good learners themselves, but actively discouraging/impeding their kids from becoming good learners)? That's a huge problem in a lot of the places where our education system struggles the most.

huntermaats5 karma

1) Katie, my co-author, is a Montessori kid and 100% credits her own academic success to the Montessori method. Specifically, she says it gave her accountability, time-management and self-teaching. Everything else followed from that. 2) Absolutely. I think we're all suffering from a generational learning apathy to some extent. Even if your parents were highly interested in, say, literature, it is unlikely that they were interested in learning everything. Our point is that this is a new era. A second renaissance. We want our students and ourselves to be open to learning anything, because innovation doesn't come from being narrowly good at one thing (Industrial Age thinking) but from being broadly informed about many things (Informational Age thinking).

IdiosyncraticOwl3 karma

[deleted]

huntermaats7 karma

1) Because learning is addictive as hell. When you learn something new, you release dopamine which is the same chemical that comes from having sex and eating anything tasty. So to me, it was mystifying as to why every kid is not addicted to learning. I think the other part is that growing up around the world, I saw that people had wildly different beliefs about what was cool but it was pretty clear that everybody thought being really, really smart was awesome. 2) Absolutely. We want to do a middle-school edition that will be a comic book. An adult edition that will deal with self-educating to stay competitive in the workforce (and dealing with your own kids). Finally, we want to do an edition for retirees looking to stay mentally active. As far as the adult edition goes, having that deep-seated confidence in your intelligence is huge. It's really no longer sustainable to place limits on yourself and say "I didn't get the math gene." You've got to be able to acquire any skill no matter what field it's in. The economy moves fast. If you aren't ready to adapt, well...you're missing out.

I'll come back to 3-7 after I've answered a few other posts.

IdiosyncraticOwl3 karma

[deleted]

huntermaats3 karma

3) Ultimately, we're going to see the flipped classroom. That means that students will be provided with sophisticated digital materials to teach themselves. The teacher's time will then be freed up to help the student figure out why their learning process is or is not working. That's the goal is to make the student effective at teaching themselves. Kids say all the time that they may "never use this stuff." In many cases, they may be right. It's a myth to say we know the skills that tomorrow's workers will need. So, yeah, it's about making the student the focus of the educational process and getting them to reflect on how they are using their own brain and learning resources. Are they using them effectively? What can they leverage them into? Essentially, it's to turn them into bootstrappers, because even if you're working for a company in the future...you essentially need to be an entrepreneur.

jurgwena2 karma

would you agree that in a world where the "right" (in the sense of economically desirable) skills at any moment presents a moving target, our schools should be focused on teaching critical thinking skills, rather than how to succeed on standardized tests?

huntermaats2 karma

Absolutely!!! There's value in learning the material, but the real value that comes from learning the material is that you are refining your approach to learning ANY material. That way, you can be flexible enough to teach yourself whatever the "right" skills of the moment are. The good news is that we don't have to wait for the schools to change. People at every age level can be giving themselves that experience. Just because the educational system is trying to get me to uncritically accept a set piece of information, that doesn't mean I can't think critically about it. Thinking critically will not only allow you to master the prescribed material faster, it'll also give you an approach to learning and problem-solving that you can apply to a constantly changing economy.

huntermaats2 karma

7) Callen deserves big credit for that. He's great. That said, I'd love to do many, many more podcasts. Katie and I have talked about doing a show of our own. Anything, in particular, you'd like to hear?

huntermaats2 karma

4) I actually think this question is an awesome opportunity to illustrate in a larger sense the power of what I'm talking about, so if you don't mind I'll go through my own experience with learning languages.

My mom is American from Kansas City. She's only ever learned one foreign language really well: Portuguese. That's because she loved Brazil. She's dabbled in many, but never really committed to them through some mix of self-doubt and not thinking its a worthwhile investment. We'd be moving, everybody spoke English etc.

My dad is Dutch. When you're Dutch, you grow up with the certainty that no one will ever learn your language so you'd better learn everybody else's. As a Swedish friend says, "Dutch isn't a language. It's a throat disease." It's pretty standard for Dutch people to speak multiple languages. At various points in his life, my dad has spoken about ten. Although he sleep talks in Dutch, his best language is actually English because that's where he has the broadest vocabulary. He left Holland when he was eighteen and his Dutch vocabulary has barely expanded since then because he has lived most of his professional and personal life in English. The exception was in Brazil.

When I was born, the prevailing wisdom was that children should not be exposed to multiple languages so my dad never spoke Dutch to me. Now, it's been amply demonstrated that there are huge cognitive benefits to being bilingual. (A good example of science-based cultural assumptions at work.) The result was that when we moved to Brazil, my parents didn't even speak to me in English. And so my first language was Portuguese. That's all we spoke at home.

When we moved to New York, one of the teachers at my kindergarten told me that only "immigrants" spoke Portuguese. Did I understand how ridiculous her comment was? No. But emotionally I understood that speaking Portuguese was--in the eyes of this authority figure--shameful. So, I refused to speak Portuguese. My mom who loved Portuguese and loved everything Brazilian would speak to me in Portuguese. I would answer in English, then eventually I just ignored her. I forgot the entire language.

By the time I finished high school, I had studied Latin, Ancient Greek, French and Japanese. I did the worst in French, because I didn't like it. Why I didn't like it who knows, but I didn't and I held onto that feeling rather than choosing to change my attitude. Japanese I did really, really well in because I had chosen the language and I've always thought Japan was a fascinating place.

So, then I went to college, and the summer after Freshman year, I worked for the Smithsonian in Panama and taught myself Spanish. Obviously, I had the benefit of plenty of opportunity to practice, but in terms of actual learning materials all I had--or needed--was three books: 501 Spanish Verbs, Essential Spanish Grammar and Joseph Keenan's Breaking Out of Beginner's Spanish. By the end of the summer, I was fluent.

(Here, I'm using fluent to mean being able to have a fluent conversation in most everyday settings. Of course, there was a lot of specialized vocabulary for say medicine or economics that I might not know. Fluent is a very messy and problematic word. By the definition I just used, I'm fluent in English and my dad is fluent in Dutch. Either way though, there are definite limitations to what I can do in English. There are areas of learning, the world etc where I don't have the vocabulary to communicate effectively in English. For example, can I have a flowing conversation with someone with a thick Glaswegian accent…probably not.)

There's two interesting sub notes to the Spanish-learning experience. 1) One of the things that gives native English speakers the most trouble when learning Spanish is acquiring the trilled or rolled "r." I was able to teach it to myself simply by doing what babies do: isolating the sound. Rather than trying to say whole worlds, I would just roll the r like a cat purring. Then, I moved into syllables following the trilled r with vowels. Then, I built up to whole words. That's the sort of specific isolated practice that Ericsson calls deliberate practice, not that I knew about that then. 2) By learning Spanish, my French improved dramatically. I was interested in Spanish and essentially by learning the grammar of Spanish, I'd finally figured out the grammar of French. I could have done that during all those years of French in my secondary education but I didn't, because I didn't like French. Probably, in part, because I didn't think I was good at it.

The summer after my sophomore year, I taught myself Italian while working for Let's Go with 501 Italian Verbs and Essential Italian Grammar.

My senior year of college, I'd finished all my biochem requirements and so I took all languages.

After I graduated from college, I wasn't sure what to do with all these languages. I enjoyed speaking them, but what was I going to do. Sit around and work on maintaining ten languages? So, I stopped and they decayed.

That all changed a few years ago, when I went to visit my parents in Libya. In general, I don't feel comfortable being in a foreign language environment and making no effort to speak the local language, so I started working on the Arabic script and acquired some basic phrases. Learning even small amounts of a foreign language is a great way to generate an instant rapport. Dunbar has even argued that that is why we evolved language. It's an excellent book if you're interested:

http://www.amazon.com/Grooming-Gossip-Evolution-Language-Dunbar/dp/0674363361

The real thing I noticed was that when I came back, I was so much more alive. It is really easy to get habituated to a place and a culture and no matter how great the place or the culture that habituation is bad news. I had to be in LA, but I wanted to find a way to capture the benefits of travel without traveling. I found that returning to language learning was the best option.

I'd also noticed that listening to a whole audiobook chapter after chapter while driving was not the best use of my attention. I did all 120 hours of Shelby Foote's The Civil War. That took months and even the book is amazing, it required more effort than it should have. So, I've started created a rotating playlist of language tracks and single chapters of audiobooks. Again, many audiobooks can be downloaded for free from the public library and websites like Librivox. Some language materials, I've gotten for free via free apps and free downloads from the library. Some, I've bought, but I consider that a worthwhile investment because I intend to be doing working on my languages continuously for the rest of my life. I limit myself to time in the car, walking to places and those spare moments when you can whip out the phone and learn a couple of words. Essentially, I've found it's a great way to take advantage of that low attention moments.

I've got about 20 that I'm working on, but they're all in very, very different states of repair. Ultimately, I'd like to reach the stage where they're all ready to go at a moment's notice, but I'm not there yet. The great news is that even in the two years that I've been doing this so far, the experience has been incredibly rewarding. Language learning has tremendous benefits. It doesn't have to be expensive. It doesn't have to consume high-quality attention. Many, many Americans are descended from multilingual ancestors. We didn't lose the language-learning gene. We lost the culture. We should get it back.

huntermaats2 karma

5) It's nice of you to say. Essentially, it's a natural consequence of a particular mindset. A few things that are particularly helpful:

a) Audiobooks. I live in LA, so I drive a lot. You can download many excellent ones for free from the Los Angeles Public Library and many other libraries around the country. They're awesome.

b) The other thing I've come to realize is that innovation comes from cross-pollinating ideas from two unrelated fields. The best known modern example is Steve Jobs and the random calligraphy class he took that became the basis for fonts in personal computers:

http://articles.businessinsider.com/2011-10-06/tech/30249828_1_college-tuition-calligraphy-adoption

That understanding of innovation radically alters one's approach to personal learning. You move from a focus on narrow specialization to being as voracious as possible and learning about as many areas as possible, because you never know how those insights will intersect. Again, that perspective leads to choices which lead to results. I know I have a lot more to learn...as some of the posters on this AMA will happily agree.

c) Much of what makes a person educated is a better set of heuristics or mental shortcuts. That's what the book all culminates in. "I got a question wrong. I broke a rule or fact. I need to identify that rule or fact and if it's a rule turn it into a recipe for action." That's a mental shortcut. Anybody who uses that doesn't have to sit there and wonder whether they got that question wrong, because they're Uncle Fred didn't get the math gene or because their teacher hates them or whether it means their mom is going to shout at them and take away the Xbox. It saves a lot of time and the more it gets internalized, the more time and energy you free up for thinking about the right things. That's one of the things I'm always on the lookout for, because if you want a shortcut to expertise in any area the biggest thing to figure out is what are the heuristics that experts use. I was lucky enough to meet the screenwriter behind one of my all-time favorite comedies of all time and the whole time I was just trying to figure out what his heuristics were by listening to him talk. If you pay attention, people reveal them all the time. Steve Jobs has one as the thesis of his Stanford Commencement Address. "Connect the dots." That's a huge one.

I use it all the time. A couple of weeks ago, I was talking to a professor from a medical school here in LA and we were having this free-wheeling conversation about his work and I connected two things and he turns to me and says, "That was wild how your mind just connected those two things." Really, I'd been sitting with one of those pieces the whole time in the back of my head looking for the connection. I just connected the dots. As we formalize and disseminate those heuristics, we're going to be stunned by what people can do...until it becomes ordinary and makes sense.

huntermaats2 karma

6) The struggles I go through when learning something are the same as everyone else. a) Self-theories. I'm not good with money. Computer science is hard. It's too late for me to be a truly great sousaphone player. Ultimately, having spent so many hours dealing with those ideas both in my own life, with students and in reading the research, I realize that worrying about those things is a colossal waste of time. So, you throw the idea out and engage your attention with the next piece of information. Maybe I didn't get the computer science gene. Maybe there's a window for sousaphone greatness and I missed it. Who cares? What can I do today and is their value in doing that anyway? (This, by the way, is why I haven't answered the language question yet. I'll come back to it.) b) Quality of attention. There's huge, huge variation in quality of attention. Last night, I ate gelato, which was delicious. But, I rarely eat sugar and so my attention is not as sharp as it can be. Sleep makes a difference. Emotions make a huge difference. Being pissed off, thinking something's a waste of time etc etc. Diet, exercise, emotional life, prior knowledge how much time you have available to think about something...all make a huge, huge difference in the quality of attention that a person can provide. Managing your own learning is about managing your attention and the things that affect it. This is why the student is and must be recognized as the center of the educational process. Teachers can do many things to engage students attention, but they will never control it. Only the individual student can do that. I've come to realize just how important it is to be honest about the quality of attention that you're capable of providing and choosing the task that best maximizes that attentional resource. (Again, this becomes relevant for the language learning question.) A big sub-issue within this is then selecting what to focus on. It's well-established that you need to understand the building blocks before you can understand the whole. That said, there's a lot more to it, but I'm going to stick to what is empirically known as a basis for building whatever consensus can be built.

Really, even self-theories, are just an aspect of attentional management. It's all attention. Where is your attention? Why is it there? Is that where you you want it to be?

IsolatedHonesty2 karma

Do you have any practical advice for how I can start my kids on the right path before they go to school. (They're two and one right ni ow)

huntermaats2 karma

The main thing is how you talk to your kids about intelligence. Never say that person is smart. Say that person must have practiced a lot. Focus your kid's attention on the process of success rather than being born with intelligence.

It's also never to early to get them thinking about the unseen changes in their brain or to get them reflecting on their process. "Oh, wow! You are so much better at that now. You couldn't do that six months ago." "Every time you learn something new, you're making your brain stronger." (Learning causes neurons to become wrapped in myelin which speeds up nerve conduction.) We describe a "bare" neuron as a dirt and myelinated neurons as a paved road. So much faster. If you can get your kids to understand that every time they learn something, they're paving those pathways and that one day they can have a superhighway then you're off to the races.

wienerdogmafia2 karma

Hunter Maats! Wishing you all the best. Was a student of yours when you started Overqualified. Think wiener dogs

huntermaats3 karma

How are you?!? What's new? What are you up to these days?

glass_houses2 karma

  1. What considerations have you explored regarding how teachers and school staff reinforce these beliefs we have about ourselves?

  2. How much of the traditional public school system structure do you think plays a role in all of this? (catering to a certain kind of learner with certain strengths).

huntermaats2 karma

1) We've met lots of teachers who think this way and lots of teachers who don't. Teachers aren't just communicating subject information, they're also communicating a view of human intelligence...for better or for worse. There's been a lot of work done on teacher expectations. If you're interested, the chapter on expectations in James Loewen's Teaching What Really Happened has a good summary of that research. 2) I think it's built into the structure of our grading system. Absolutely. Having a curve says that skills are scarce. By definition, it says we can't all be good at Spanish, Math, English or Physics. Only a few of us can. That's ridiculous. Your ability to use a comma correctly does not diminish my ability to use it correctly. It should enhance it, because now you can explain the rules to me.

TishraDR2 karma

My daughter cannot understand math, period. She's going to college, but even remedial math is hard for her. She constantly asks her brother, who is a math major, to help her. I don't believe there is a gene thing, she does, however. Both her father and I were and are good in math. When she was in high school we put her Sylvan Learning for a year just work on her math skills. While she did good at there, as school she was still in the same position. She says math is like Greek to her, she just doesn't get it. What can she do to improve herself?

huntermaats2 karma

Part of the problem is that we tend to talk about being "bad at math" which is so vague that it's not a very useful piece of feedback. The number one thing I'd tell her to do is get very specific about what she doesn't get. What's not working. Then fix those specific issues one at a time.

I'd also be honest with her. If she really does believe she's genetically doomed to fail at math, then how engaged is her attention?

Generally speaking, much of what you're really dealing with when students struggle in math is fear. When we say "I was so scared I couldn't think"...that's true. The amygdala, a small almond-shaped organ in the brain, fires up and shuts down. People who are afraid temporarily shut down their higher cognitive functions. The result is that kids can sit through an entire class and receive no information. Hence your daughter saying "It's all Greek to her."

How do you beat fear? Well, first, you have to correctly diagnose the problem. Peanut butter getting stuck to the roof of your mouth isn't scary. You're scared of peanut butter getting stuck to the roof of your mouth. She should make fun of her fear of math. It is ridiculous to be afraid of squiggles on a piece of paper. What's the worst that happens? She does the same way, she's done in the past. I understand how much rides on your daughter going to college, so I'm not saying it doesn't matter. It does. I'm just saying that there may be a perspective that allows her to get past her fear and dig in. Ultimately, fear is beaten by confronting it. Identifying the specific things she doesn't understand will allow her to provide targeted fixes. I can dwell in the idea that I am "bad at math" forever. It doesn't need to take that long to figure out what the word exponent means.

Specifically, it's worth remembering that math is just rules. Rules can be clear enough to act on and they can be fuzzy. You look at a parking sign and for the first few seconds you don't know what to do. Can I park here or not? You then sit there until you have a "recipe for action." At this time on a Tuesday, I can park here. That's all algebra is: a bunch of parking signs. For every piece of algebra, you need to clarify each rule until you've answered two things: 1) what do I do? 2) when do I do it? In the book, we use the analogy of Betty Crocker. She takes a recipe and distills it down into three easy steps. That's possible for anything in algebra. You can feel however you want about parking and baking a Betty Crocker cake, but there's no doubt that they're doable. You can do this. Beat algebra into the ground one rule at a time.

jurgwena2 karma

Are you familiar with a guy named John Taylor Gatto? He won multiple Teacher of the year awards in NY and claims he did so by breaking the law. One of the lines from a lecture he gave is something like "genius is common as dirt." He also says that our educational system has been built from the ground up to churn out mediocre minds.

huntermaats6 karma

Know of John Taylor Gatto. As for the idea that our educational system has been designed to turn out mediocre minds, it wasn't just Gatto that said it. Woodrow Wilson stated that as the educational system's goal over 100 years ago, “We want one class of persons to have a liberal education, and we want another class of persons, a very much larger class of necessity in every society, to forgo the privilege of a liberal education and fix themselves to perform specific difficult manual tasks.” (Pg. 208 Lies My Teacher Told Me.)

tautologies2 karma

It has a name. Learned helplessness.:)

huntermaats3 karma

And Learned Helplessness is its name-o!

sittinduck1 karma

Hey Hunter. It's so interesting to hear your opinions on education. I am sixteen and I go to a project based learning charter school. (You actually might have heard of it) As a result I have had a hugely different experience than most kids. I love literature and I am a huge fan of reading. Most kids my age don't get it and brag about how they don't. My generation seems to cultivate this idea that willful ignorance is cool. I completely agree with you about needing a cultural change because I experience it every day. Basically my question is how can you change that idea in 16/17/18 year olds? I can definitely see how to change it in younger generations but how can you implement a complete change in a teen who is already convinced that learning things is stupid and not knowing what's going on in the world is cool? By the way I apologize for any heinous grammar or spelling errors. I have web traveling nonstop for 20 hours.

huntermaats2 karma

What's the name of the school?

huntermaats2 karma

Hey, sittinduck! So, so glad to hear from you. I'm really glad you've had such a great educational experience. Now, how do we give it to everyone else?

16/17/18 is really old. The good news is you can teach even older dogs new tricks. Everybody's first reaction to reading the book is "I wish I'd known this earlier" but pretty soon they just start applying. We've had grandparents buy our book for the grandkids--end up reading it themselves--and end up applying it in their own lives.

Even the most disgruntled teen would love to be smart. They would love to be doing well in school. The "I don't care" thing is just a way to protect themselves from that feeling of stupid. We've worked with some hard core school haters. You chip away at those beliefs about themselves and you get them to realize that they can do well. Pretty soon, the dopamine kicks in and you're off to the races. That's the chemical that's released when you learn something new. It's also the chemical that's released when eating tasty food. The thing is that dopamine can't beat the "feeling of stupid." And that's the real problem. Stupid is a feeling. It's the feeling of shame. And the feeling of shame is about the worst.

That's the big difference. You view mistakes as natural. (I'd bet.) Most kids view them as a reflection of their inherent intelligence. They feel stupid all day and they tell themselves they don't care, so they don't have to feel that way.

The good news is that you're on the right side of history. Once they realize that feeling stupid is not a fact. It's a feeling that comes from a faulty understanding of intelligence. Once teenagers or anyone else understands that, they stop doing it. Then, they love learning. It's such an obviously better experience of life that once people have it presented to them that way, they inevitably choose it.

kolm1 karma

You will love this study. There actually several which point out similar cultural links. A prominent theme are stereotyping effects. I was in the privileged bracket (white male, solid family background) and got a PhD in math easily, but as a black women from a single teenage mum, I probably had never believed I could be good at math, and not even tried.

I think one of the main points you do not touch is that we are imposing currently an as-of-yet-unheard-of torrent of 'knowledge' thrown at our kids, with little to no time for them to actually think about it (like Mr. Franklin or Mr. Leibniz or Mr. Leibowitz or any number of other people could allow themselves), and we rate them on the basis of how quickly they can re-arrange these puzzle pieces thrown at them in second intervals.. This is because life is assumed to be very complicated (which it isn't unless relationships) and complex (which it absolutely is), and children must be "prepared for life", meaning they need to learn everything we learned, plus all new knowledge which came up right now. This is unsustainable.

The longer I live, the more I get convinced that we should follow Thoreau's maxime to 'simplify, simplify, simplify', and that should start at school. The first and last and most important thing everybody must learn is how to learn, and that to stop learning means to die inside. And you must make problems your own if you want to master them. You must feel that the answer is important, for whatever reason, that it matters what you think about it.

At the end, people will discover that at least science is not populated entirely by geniuses, but by a lot of common folks as well, and that everything can be broken down to digestible bites if you have the time and the will to chew on them a little. College math in particular is much more about patience and diligence than brilliance.

huntermaats3 karma

"The first and last and most important thing everybody must learn is how to learn, and that to stop learning means to die inside. And you must make problems your own if you want to master them. You must feel that the answer is important, for whatever reason, that it matters what you think about it."

Well said!

Being able to remove yourself from the noise, so you can fill your life and your mind with only what's worthwhile is essential.

run_zeno_run1 karma

Carol Dweck has done great research about this and wrote a terrific book called Mindset. To summarize briefly, there are two mindsets a child (or adult) can have about their abilities, a fixed mindset and a growth mindset. Rewarding people for high intelligence and being smart, or the opposite, of having low intelligence and not being smart, reinforces a fixed mindset where a person's abilities are predetermined and innate. Rewarding people for their practice, effort, hard work, and determination to master a subject reinforces a growth mindset. Almost across the board, successful people have a growth mindset.

My first question is: how would you change education, in specifics please, to incorporate the findings of Dweck and instill a growth mindset in learners.

Also, Peter Thiel has said that the modern educational system is a competitive battleground played out by the students mainly for insurance and status, to provide security in the competitive job market. Higher education exploits this by charging more and more for their elite certifications, while public schools contribute by emphasizing weeding out students based on simplistic metrics of performance. He thinks startups that are trying to disrupt the education market cannot avoid understanding this and must figure out how to provide the insurance and status currently monopolized by those incumbent institutions.

So my second question is: While it is well and good to focus on helping students do better in class using neuroscience and psychology, this still leaves the systemic problems of the educational system itself in place. Do you have any thoughts on how to change the institutions to help students become better explorers and creators instead of just industrialized employees?

Thanks!

huntermaats4 karma

I'm going to answer your questions in reverse order.

2) Peter Thiel is right. There are two parts to the educational system: 1) skill-acquisition and 2) accreditation. Here's why I think Thiel is wrong. Accreditation is really expensive and the expense is only justified because employers buy into the myth that surrounds graduates of fancy institutions. They believe that you get more value from somebody with a fancy degree. Essentially, that rests on the assumption that a person with a fancy degree is smarter. They're not. At least, not inherently so. The real problem is we don't measure absolute performance. Can you use a comma correctly or not? We only know how to measure relative performance. Can you use a comma correctly more often than your classmates? Create a system that meaningfully measures actual skills and at a certain point the market will take over. Employers will demand that applicants demonstrate their skills. That will disrupt the whole process of accreditation. Suddenly, I have a choice between paying a guy with a fancy degree and a guy I can pay much, much less who has the same or better skill set. At that point, people won't pay for the fancy degree. Of course, some skills we can't measure. Entrepreneurship, resourcefulness, innovation etc...but let's be honest about the fact that we can't measure them rather than saying that college admissions offices can. An educational start-up that can give employers meaningful absolute performance data will disintermediate an educational start-up that can only give employers averaged-out relative performance data. That also goes for most individuals. They don't know how to make sense of intelligence, so that let Universities make that determination for them. It's a terrible, terrible system, because you end up with less competition in the labor market and a bifurcation in the intellectual life of the culture. There's the pro-fancy University crowd and the anti-fancy University crowd. One unthinkingly accepts the ideas of the fancy professors and one unthinkingly rejects the ideas of the fancy professors. That is really bad news.

So, essentially, Thiel worrying about the Universities is like Thiel worrying about Tower Records. Oooh! Scary. As people start measuring skills, employers will stop paying for degrees because it's bad value for money. With the insurance effect of a University degree gone, the Universities will either have to figure out some sort of value add or they will go the way of the record shop. Harvard is a strong brand, but so was Tower Records.

What needs to be done at the University level is the same thing as what needs to be done at every other level move the focus from teaching to self-teaching. Get the students to be responsible for their own learning and then step in when they run into trouble or to provide enrichment. Also, I think universities can learn a lot from Silicon Valley. There is zero reason why students shouldn't be doing start ups within schools. If you want to be a novelist, then start writing and publishing in college. Frankly though, this could be happening in high school. There should always be a blend between acquiring abstract knowledge and attempting to apply it in the real world.

huntermaats2 karma

1) It's all about moving the focus from teaching to self-teaching. Students should be provided with the materials and should be responsible for teaching themselves. When that's not working, that's when the coach steps in and helps the student diagnose their own process and figure out what's going wrong.

In addition, that practical component should be combined with a knowledge of how the brain learns. Essentially, there are really three things you need to understand in order to learn anything: automaticity, attention and emotions. Automaticity just means that if you do anything often enough, it becomes automatic. What you apply your attention to is what becomes automatic. Your emotions direct your attention. Being able to effectively diagnose what is working, what is not working and how to solve it is the key to not only being an effective learner but also to entrepreneurship. That metacognition--or thinking about thinking--is what the National Academy of Sciences identified as the hallmark of effective learners.

The classroom I'm talking about is called the flipped classroom. I'd also throw out grades and move to measuring skills. (See my answer to your second question.) Basically, the educational system would present students with materials that they could use to acquire skills. Teachers would be there to help diagnose and overcome roadblocks in that process and provide enrichment. I would also say that kids should be encouraged to compete in the marketplace from day one. That's where you really discover the shortcomings of your skills.

Greasytoes1 karma

Your basic thesis reminds me of Robert Anton Wilson a lot, a writer and thinker I truly love. So my question is: Have you ever or would you ever do acid? Also: I know Timothy Leary is really disreputable, but what's your opinion on his theorem concerning brain circuits, and do you hate him as an educator for his part in demeaning education in the 1960s? Thank you for your time, very interested in reading your book!

huntermaats2 karma

I don't know much about Tim Leary's theories on brain circuits. We went off peer-reviewed scientific journals. Tim wasn't in there. That said, I don't think I hate Tim Leary. Should I?

I've never done acid. I don't even drink. So, I doubt I'll be doing acid in the future.

That said, I love to try new authors. Is there are any of Robert Anton Wilson's writing you'd particularly recommend?

BrandonLomar1 karma

Thoughts about Dunning Kruger effect?

http://en.wikipedia.org/wiki/Dunning%E2%80%93Kruger_effect

huntermaats1 karma

Ah, the Dunning-Kruger effect aka misplaced confidence aka arrogance. Self-confidence and performance don't always correlate, though there is some evidence that they correlate much better in girls than it does in boys.

In general, self-assessment isn't very reliable, which is why you have tests to give you realistic and impartial assessment. More specifically, it's really important that the tests give specific feedback about exactly which aspects of the subject you don't understand. A "A" or "B" or "F" aren't very informative. They tell you nothing that is actionable. On the other hand, if the test tells you that you don't know the "power rule" or how to use a specific type of comma...that is actionable. I now know what I need to fix and the feedback is specific enough that I can't really deny the need to fix it.

crankypants_mcgee1 karma

Oh Hunter, that would mean people on a large scale taking personal responsibility for their children and to a certain extent other people's. The way we are ruining our country now we get to bitch that it's all the politician's fault and still go shopping at Wal-Mart 3 times a week!

huntermaats6 karma

Hey, crankypants_mcgee. Love the screen name. It'd mean a lot more personal responsibility for all of us. Both the Republicans and the Democrats are trying to sell us on the myth that what's wrong with this country is the other 50% of the population. Of course, our politicians aren't the best, but in a democracy whose fault is that if not the people's?

bigbluemarble1 karma

[deleted]

n5corp1 karma

Read through the other responses, the concepts can be applied to any kind of education.

huntermaats2 karma

As n5corp said, the point is that all education works in the same way. We've come to think of it as different, but the underlying process for different subjects and different levels of complexity is the same. To see if the book would be useful for you, you might want to check out the following free chapter:

http://thestraightaconspiracy.com/wp-content/uploads/2013/02/Straight-A_Chapter7_FreeDownload1.pdf

Creabhain1 karma

What do you think of the guy who plans to become a golf pro through 10000 hours practice?

Here is a quote from his site

Talent has little to do with success. According to research conducted by Dr. K. Anders Ericsson, Professor of Psychology at Florida State University, “Elite performers engage in ‘deliberate practice’–an effortful activity designed to improve target performance.” Dr. Ericsson's studies, made popular through Malcolm Gladwell's book Outliers and Geoff Colvin's Talent is Overrated, have found that in order to excel in a field, roughly 10,000 hours of "stretching yourself beyond what you can currently do" is required. "

huntermaats2 karma

I say go for it. The key thing is making sure he's doing that deliberate practice that Dr. Ericsson is talking about. I wonder how many hours Tiger has done?

econguy1 karma

changing our culture.

By the Mystical Powers and Random Chance.

You don't ask for much do you? Though you and I are on the same page on this point, culture tends to drive the success of an education system more than anything else. I will read your book (though it will probably preach to the choir) you have a supporter in me by default if your movement gathers speed.

huntermaats1 karma

Mystical powers and random chance would be nice, but I think we can do it with more terrestrial means. Cultures are made of individuals. Individuals can change their minds. Why can't we change the entire culture? Mainly because we've gotten comfortable with the idea that cultures have a life of their own. I say we can shape it. If I'm wrong, I'll buy you a drink.

wwwwho1 karma

[deleted]

huntermaats5 karma

Blowing smoke up someone's ass sounds pretty unpleasant for blower and blowee. The self-esteem movement has been a huge failure. Our point is not to tell each child that they are a rare and special flower, but rather to tell them you are an ordinary human being. And just like every other ordinary human being, if you work hard you'll get results: definitely academic, probably financial and definitely in terms of self-respect and dignity. Every kid should have self-esteem, but they're not going to get it from a trophy for participation. They're going to get it the same way everybody else does: from working.

CommentsRarely1 karma

Great reply regarding the "self-esteem" movement. I couldn't agree more.

huntermaats2 karma

Thanks, CommentsRarely. Glad to have gotten one of those rare comments.

_MUY1 karma

[deleted]

huntermaats1 karma

Hollywood's ability to explain every superpower as being a result of "genes" is pretty telling. Genes have become a magical explanation for everything.

I'd love to see a decent lab program in schools. If students can be teaching themselves the material, that would free up resources--time and money--for more lab bench work.

philosophical_whale1 karma

In most of my high school classes I find that other students who are on the lower end of the grade spectrum do indeed follow what you describe. I find that the solution lies within motivation. If you can get kids to have self confidence/the willingness to perform then this barrier can be broken.

Although you have discovered the true face of the problem, how can you deal with the root of it? The motivation aspect? You cannot simply say try harder, the "math gene" is merely an excuse for kids to be lazy, give up, and not rise to the occasion. This is the same in sports as well, no one can tell you that you are athletic or not. You tell yourself that, granted that some are physically impaired.

How exactly would you deal with this motivation problem on a widespread basis?

huntermaats1 karma

The point is what is the source of that lack of motivation. We now have a lot of research to say that those beliefs are the source of that lack of motivation. In other words, believing you didn't get the "math gene" comes first. Lack of motivation comes second. There's been a lot of studies done where by changing students' beliefs about their intelligence--and I've found this with my own students and early feedback on the book--that motivation then kicks in. Change what we believe about intelligence and the motivation will follow.

So, really, the question is how do you change a belief: one person at a time.

somatrip1 karma

I am sure it has been brought to your attention before but I recall a study that showed that students that were praised for their intelligence after doing well performed worse in the long run than students that were praised for their hard work. Essentially students in the first group learned to attribute their success to something out of their control so their ability work hard to overcome obstacles suffered as a result. The study seems to really fit in with what youre talking about.

edit: found the study by carol dweck; http://nymag.com/news/features/27840/

huntermaats2 karma

Dweck's work is pretty much the cornerstone of what we're talking about. It all fits within the much larger context of Seligman's work on Learned Helplessness. Essentially, do people believe that they control their educational destiny or not? Great article. Thanks for sharing.

Dweck has also shown that sharing with students knowledge of how the brain works can displace those beliefs. Check out Mindset Works to see what she's doing on that front:

http://www.mindsetworks.com/default.aspx

mmangino2 karma

Yay for bringing up Seligman. I've really enjoyed his work.

huntermaats2 karma

It's incredible stuff. I wish more people knew about it.

SqueakyTiki1 karma

I think you may be onto something here.

huntermaats1 karma

It's certainly made a big difference for everyone we've talked to so far.

PlasmaYAK1 karma

First of all thanks for doing this! I don't believe in there being a gene for certain intelligence, but I know it's a lot harder for me to study something I have no interest in. I'm currently at university and my general chemistry class is such a pain to me, for all I want to do is learn more about computer science. Anyways i'm curious as to what your books are doing to help students become not interested in just learning, but in learning a wide variety of fields?

huntermaats1 karma

You've identified the issue. It's emotional. You feel differently about chemistry and you think of it as a chore. I guess the question is what is chemistry going to do for you? Likely, you see computer science as relevant to your personal goals and chemistry...not so much? The key is finding ways that chemistry fits into the big picture. Some ideas:

1) Required for your degree. 2) Could you use your dislike of chemistry to develop apps for learning chemistry that are so fun, chemistry-haters everywhere would want them? 3) Are you interested in building hardware? In that case, knowledge of chemistry would probably be beneficial.

In general, the subjects you're not interested in are a great opportunity for you to develop your imagination. You are forced to figure out how it can or would be relevant. We all want to keep learning in the areas in which we're comfortable, but innovation comes from cross-pollinating in different fields. If you just keep learning computer science, you'll probably come up with similar solutions to what other computer scientists have already come up with. My guess is you're looking to do something big and different, right? Have you read the Steve Jobs Stanford Commencement speech? His big idea on what a personal computer could be didn't come from a computer class.

http://articles.businessinsider.com/2011-10-06/tech/30249828_1_college-tuition-calligraphy-adoption

Connecting the dots between chemistry and computer science is more likely to give you that big idea than just doing computer science. People often forget that Zuck was also a psych major.

pundemic1 karma

How do you and your father feel about geese?

huntermaats7 karma

We don't trust 'em. Shifty-eyed descendants of dinosaurs that they are. That said, we feel very good about them when roasted. Dangerous as predators, but geese are delicious as prey.

RedditSucksRecently0 karma

Your misunderstandings of culture and the process of socialization are breathtaking. Culture is self-perpetuating, and cannot be manufactured or changed (instantaneously) by a single person. There are marketing companies who attempt to do this, but predicting the success of such ventures is incredibly difficult. Not to mention, environment is not the only determining factor in how a person turns out. This is why all social sciences are constantly developing new theories on the mechanisms that drive society.

While you are correct in your statement that the current status of the US public education system is somewhere between "in total disarray" and "an absolute fucking disaster", your statement that we should simply "change our culture" defies the characteristics of culture itself. That is, culture is intangible, and it is difficult to know what will and will not affect it, and what the result of that effect will be.

Take language for example, one of the earliest institutions that lead to the development of larger communities and the development of early culture: It is beyond obvious that language varies, and even within particular languages the dialects of languages that are spoken vary from place to place. This is an excellent example of the unpredictability in regards to culture. That is, because it is the result of crowd-sourcing, the great amount of variation among people can lead to one thing going in many different directions (Just as French and Spanish are descendants of Latin).

This is where the line between Sociology and Psychology must be drawn, as the impacts had on such things must at some point come down to a personal level. People have different tendencies toward different things. I myself used to be a "Math Guy", but regardless of my ability to do math, I simply despise its seemingly tedious nature, and the kind of people it brings me in contact with. While we all may have the same potential to be "Math Guys" it is impossible for us all to be one simply due to personal preference.

I found myself wondering just today, "What kind of accident must have led to the development of the game Horseshoes?" Further, "How long did it take to actually work out the rules? Do today's rules reflect the original rules?" The bare bones of the game reflect the creators' ideas of fun, fairness, and competition; Rules which will vary depending on where you play the game, because (depending on region) the rules have all passed through different hands.

Perhaps this is a ramble, and perhaps I'm misinformed on what you mean by "change our culture", as I have only read your title, and the body of your text here on Reddit. I was so taken aback by the idea of simply "changing the culture", however, I figured I'd just lay out why it's more complicated than that.

huntermaats1 karma

I'm sorry to hear that Reddit Sucks Recently, RedditSucksRecently. What was it like in the glory days?

You can change culture and we have. See hand-washing and the germ theory. There is a belief about disease that has been widely institutionalized and an effective practice that has been adopted by many, many people. (Unfortunately, not all.) What we're trying to do is the same displace an old model of intelligence and institute behaviors--looking up things you don't know, analyzing your mistakes etc--that have been empirically proven to improve outcomes.

We've done it before and we can do it again. I've already offered to eat a hat and buy someone a drink if I fail, what would you like?

smcdark-2 karma

would you rather fight a horse-sized duck or a hundred duck-sized horses?

huntermaats4 karma

Duck-sized horses. Get on some sort of elevated platform and you can pick the my little ponies off at will. Flying totally changes things in the duck's favor.

Fapologist-2 karma

I have a very important question if I can get your attention.

huntermaats3 karma

You'll get my attention. Go right ahead.

Fapologist-8 karma

Would you rather fight one horse sized duck, or one hundred duck sized horses?

huntermaats2 karma

A hundred duck-sized horses. I fear the bills.

efluermoli-2 karma

[deleted]

busted_up_chiffarobe1 karma

Exactly. Selling something.

huntermaats5 karma

I am selling something: an idea. I'd be perfectly happy if nobody bought the book, if we could live in a world where everybody was woken up to just how much people are capable of achieving. I totally believe that this is what is needed to fix our country and empower kids everywhere to better their circumstances.

watersign-9 karma

Gonna have to disagree with you here bud, asians are good at math because their language is very complex and the way their brains are formed as youngins' helps them grasp math concepts better than most.

Jews also tend to score Higher on IQ scores relative to the size of their population. I know you mean well, but like you said..education is taken, not givin rings pretty true. We'd be better off getting rid of schools altogether and just keeping them open to people who actually want to go to school and learn useless knowledge like most of what is taught today in the American school system.

America needs to make its education system like Germany, where they take the smart ones and put them on the college path, take the 'not-smart' ones and show them how to use tools to fix stuff so they don't wind up with liberal art degrees and manning star bucks registers with 250K in debt.

Unschooling will become more popular in the near future because people will start to wake up and realize what a sham formal education is, atleast in America.

buppers4 karma

intelligence isn't genetic. there's a ton of research on this.. And to decide at a young age that someone is unintelligent and destined to be a cashier or landscaper or something is repugnant.

huntermaats5 karma

Agreed. To decide at a young age what anybody should be is repugnant. That's for the individual to choose for themselves.